Difference between revisions of "2009 AMC 10A Problems/Problem 7"

(Solution)
(Problem)
Line 4: Line 4:
 
<math>
 
<math>
 
\mathrm{(A)}\ \frac{12}{5}
 
\mathrm{(A)}\ \frac{12}{5}
\qquad
+
\qq\ 42
\mathrm{(B)}\ 3
 
\qquad
 
\mathrm{(C)}\ \frac{10}{3}
 
\qquad
 
\mathrm{(D)}\ 38
 
\qquad
 
\mathrm{(E)}\ 42
 
 
</math>
 
</math>
  

Revision as of 15:28, 16 April 2021

Problem

A carton contains milk that is $2$% fat, an amount that is $40$% less fat than the amount contained in a carton of whole milk. What is the percentage of fat in whole milk?

$\mathrm{(A)}\ \frac{12}{5} \qq\ 42$ (Error compiling LaTeX. Unknown error_msg)

See also

2009 AMC 10A (ProblemsAnswer KeyResources)
Preceded by
Problem 6
Followed by
Problem 8
1 2 3 4 5 6 7 8 9 10 11 12 13 14 15 16 17 18 19 20 21 22 23 24 25
All AMC 10 Problems and Solutions

The problems on this page are copyrighted by the Mathematical Association of America's American Mathematics Competitions. AMC logo.png